解析力学/ラグランジアン のバックアップソース(No.3)

更新

&katex();

* 目次 [#z557608a]

#contents

* ラグランジアンの定義 [#oa193382]

力学系の運動エネルギーを $T$、ポテンシャルエネルギーを $U$ とするとき、
ラグランジアン $L$ を、

$$
L=T-U
$$

として定義する((もっと柔軟な定義もあるのだけれど、ここでは話を簡単にするためこの狭義の定義で話を進める。))。

* 力学系の座標と速度 [#a3b96963]

その力学系を記述するための座標が $q_1,q_2,\dots,q_n$ で表されるとしよう。

>3次元中での質点1個の運動なら $n=3$ で、$q_1=x,q_2=y,q_3=z$ と考えればいい。

>3次元中での質点2個の運動なら $n=6$ で、$q_1=x_1,q_2=y_1,q_3=z_1,q_4=x_2,q_5=y_2,q_6=z_2$ と考えればいい。

>質点1が原点から距離 $r$ の球面上を動くなら、球座標を使って $n=2$ として $q_1=\theta,q_2=\phi$ と取れば良い。

また、それらの座標の時間微分を $\dot q_k=\frac{dq_k}{dt}$、 $\ddot q_k=\frac{d^2q_k}{dt^2}$ などと表すことにする。

一般に運動エネルギーは $q_1,q_2,\dots,q_n$ および $\dot q_1,\dot q_2,\dots,\dot q_n$ の関数となる。またここではポテンシャルエネルギーは $q_1,q_2,\dots,q_n,t$ の関数としておく。

$$
L(q_1,q_2,\dots,q_n,\dot q_1,\dot q_2,\dots,\dot q_n,t)=T(q_1,q_2,\dots,q_n,\dot q_1,\dot q_2,\dots,\dot q_n)-U(q_1,q_2,\dots,q_n,t)
$$

カーテシアン座標系($x,y,z$座標系)を思い浮かべていると運動エネルギーに $q_1,q_2,\dots,q_n$ が含まれるのはおかしいと感じるかもしれないが、例えば極座標では運動エネルギーに $r$ や $\sin\theta$ が現れる。

ローレンツ力などでは速度に応じた力が生じるため上記のようなポテンシャルでは表すことができない。そのような場合については後ほど個別に考える。
* 最小作用の原理 [#maf140c9]

時刻 $t_1$ の座標値 $q_k(t_1)$  と、時刻 $t_2$ の座標値 $q_k(t_2)$ が与えられたとすると、
その間での系の運動 $q_k(t)$ は「作用」と呼ばれる値

$$
S=\int_{t_1}^{t_2}L\ dt
$$

を最小にするようなものとなる(ラグランジアンの積分値として与えられる)、というのが最小作用の原理である。

何を言っているかというと、各座標の最初と最後の値が決まってもその間で座標が時間と共にどのように変化するかはいろんな可能性が考えられる。しかし、そのうち実際に起きる運動は $S$ を最小とするものになる、ということだ。(一般に運動方程式は時間に対する2階の微分方程式であるから、1つの座標につき2つの自由パラメータを持つが、始点と終点という2つのパラメータを指定すると運動は1つに定まる(初期座標と初期速度を与えるのと同じ))

え、そんな話は聞いたことがないんだけど、と思うだろうけど実際こういう原理が存在するのだから仕方がないと諦めよう。上記の話とこれまで学んできたニュートン方程式が成り立つ話とが矛盾しないことを以下で見る。

* ラグランジュの運動方程式 [#x337268d]

「作用を最小化する」という条件を微分方程式の形に直したものをラグランジュの運動方程式と呼ぶ。

ある座標の軌跡 $q_k(t)$ を少しだけ変化させて $q_k(t)+\Delta q_k(t)$ としたとする。$q_k(t)$ が最小作用を与えるという意味は、どのように $\Delta q_k(t)$ を選んだとしても $S$ は元の値より増加する、ということと同義である。ただし始点と終点は決まっているので $\Delta q_k(t_1)=\Delta q_k(t_2)=0$ とする。

** 汎関数微分 [#u34af725]

$S$ は関数 $q_k(t)$ を与えると値が決まるという意味で「汎関数」と呼ばれる。$S$ が $q_k(t)$ の汎関数であることを強調する際には

$$S=S\big[q_k(t)\big]$$

などと書く。$S$ の $q_k$ による「汎関数微分 $\frac{\delta S}{\delta q_k}$」は、

$$
\begin{aligned}
\Delta S&=S\big[q_k(t)+\Delta q_k(t)\big]-S\big[q_k(t)\big]\\
&\sim\int_{t_1}^{t_2}\frac{\delta S}{\delta q_k}\Delta q_k(t)\,dt
\end{aligned}
$$

として定義される。

普通の関数 $f(x)$ の微分が

$$
\begin{aligned}
\Delta f&=f(x+\Delta x)-f(x)\\
&\sim\frac{df}{dx}\Delta x
\end{aligned}
$$

であるのと比べると、汎関数微分の定義になぜ積分が現れるのか疑問に思うかもしれないが、その疑問は、多変数関数 $f(x_1,x_2,\dots,x_n)$ の偏微分が、

$$
\begin{aligned}
\Delta f&=f(x_1+\Delta x_1,\ x_2+\Delta x_2,\ \dots\ ,x_n+\Delta x_n)-f(x_1,x_2,\dots,x_n)\\
&\sim\sum_{k=1}^n\frac{\partial f}{\partial x_k}\Delta x_k
\end{aligned}
$$

であることと比べると解決するだろう。汎関数 $S$ は $t_1$ から $t_2$ までのすべての時刻に対する $q_k(t)$ の値を引数とする超多変数関数であると見做せるのだ(個数の引数は連続無限)。それら個々の値の変化に、その値に対する微分値 $\frac{\delta S}{\delta q_k}$ を掛けて足したものが全体としての変化となる。

ということで、当然だが汎関数微分 $\frac{\delta S}{\delta q_k}$ は時刻の関数となる。

** 最小点においては汎関数微分がゼロとなる [#ifcb74cb]

$f(x)$ の極大点・極小点において $\frac{df}{dx}=0$ が成り立つのと同様に、最小作用を与える $q_k(t)$ に対して、

$$\frac{\delta S}{\delta q_k}=0$$

が成り立つ必要がある。

実際に汎関数微分を計算してみるにあたっては $q_k(t)$ を変化させると $\dot q_k(t)$ も変化することを加味する必要がある。さらに $\Delta q_k(t_1)=\Delta q_k(t_2)=0$ を用いた部分積分を適用すると、

$$\begin{aligned}
\Delta S
&=\int_{t_1}^{t_2}\bigg[\frac{\partial L(\dots,q_k(t),\dot q_k(t),\dots)}{\partial q_k(t)}\Delta q_k(t)+\frac{\partial L(\dots,q_k(t),\dot q_k(t),\dots)}{\partial \dot q_k(t)}\Delta \dot q_k(t)\bigg]\,dt\\
&=\cancel{\bigg[\frac{\partial L(\dots,q_k(t),\dot q_k(t),\dots)}{\partial \dot q_k(t)}\Delta  q_k(t)\bigg]_{t_1}^{t_2}}+\\
&\phantom{=}\int_{t_1}^{t_2}\bigg[\frac{\partial L(\dots,q_k(t),\dot q_k(t),\dots)}{\partial q_k(t)}\Delta q_k(t)-\frac{d}{dt}\bigg\{\frac{\partial L(\dots,q_k(t),\dot q_k(t),\dots)}{\partial \dot q_k(t)}\bigg\}\Delta q_k(t)\bigg]\,dt\\
&=\int_{t_1}^{t_2}\underbrace{\bigg[\frac{\partial L(\dots,q_k(t),\dot q_k(t),\dots)}{\partial q_k(t)}-\frac{d}{dt}\bigg\{\frac{\partial L(\dots,q_k(t),\dot q_k(t),\dots)}{\partial \dot q_k(t)}\bigg\}\bigg]}_{=\,\frac{\delta S}{\delta q_k}}\Delta q_k(t)\,dt\\
\end{aligned}$$

を得る。ここで汎関数微分 $\frac{\delta S}{\delta q_k}$ をゼロと置くと以下を得る。

$$
\frac{d}{dt}\bigg\{\frac{\partial L}{\partial \dot q_k}\bigg\}-\frac{\partial L}{\partial q_k}=0
$$

この方程式はラグランジュの運動方程式と呼ばれ、任意の $k$ に対してこの式が成り立つことが「停留作用」の法則の微分形を与える。この方程式に現れる $\frac{\partial L}{\partial \dot q_k}$ や $\frac{\partial L}{\partial q_k}$ は、$\dot q_k$ と $q_k$ とを独立の変数とみなして偏微分したものであることに注意せよ。

ラグランジュの運動方程式自体は作用が最小となることを保証しない。作用が停留値を取ることのみを与える。ラグランジュの運動方程式を解いて得られた軌道の近傍で小刻みに振動するような軌道を考えれば、ポテンシャルエネルギーをほとんど変えないまま運動エネルギーを増加させることがいつでも可能である。したがって、求まる停留値が極大値である可能性はない。ただしそれが鞍点ではないのか、あるいはもし極小値であったとしてもそれが真の最小値であるのか、についてはここまでの議論からは判然としない。

** ニュートン方程式と矛盾しないこと [#a6468f36]

ポテンシャルエネルギー $U(x,y,z)$ 中で運動する質量 $m$ を持つ質点の運動は、ニュートン方程式

$$
\begin{cases}
m\ddot x=-\frac{\partial}{\partial x} U\\
m\ddot y=-\frac{\partial}{\partial y} U\\
m\ddot z=-\frac{\partial}{\partial z} U\\
\end{cases}
$$

で記述される。(力は $\bm f=-\bm \nabla U$ で与えられることを思い出せ)

一方、この系の運動エネルギーは $T=\frac{1}{2}m(\dot x^2+\dot y^2+\dot z^2)$ であるから、

$$
L=T-U=\frac{1}{2}m(\dot x^2+\dot y^2+\dot z^2)-U(x,y,z)
$$

である。

ラグランジュの運動方程式は、

$$
\begin{cases}
\displaystyle\frac{d}{dt}\bigg\{\frac{\partial L}{\partial \dot x}\bigg\}=\frac{\partial L}{\partial x}
\ \ \to\ \ \frac{d}{dt}(m\dot x)=-\frac{\partial}{\partial x}U\\
\displaystyle\frac{d}{dt}\bigg\{\frac{\partial L}{\partial \dot y}\bigg\}=\frac{\partial L}{\partial y}
\ \ \to\ \ \frac{d}{dt}(m\dot y)=-\frac{\partial}{\partial y}U\\
\displaystyle\frac{d}{dt}\bigg\{\frac{\partial L}{\partial \dot z}\bigg\}=\frac{\partial L}{\partial z}
\ \ \to\ \ \frac{d}{dt}(m\dot z)=-\frac{\partial}{\partial z}U
\end{cases}
$$

となって、確かにニュートン方程式と一致する。

実はラグランジアンや作用を適切に定義することにより、
ニュートン方程式だけでなくマクスウェル方程式や一般相対論などについても
最小作用の形で定式化できることが知られている。



* 例題 [#qd0685db]

ラグランジュ方程式がニュートン方程式を与えるだけであるなら、なぜ新たにラグランジュ方程式を考える必要があるのだろう。その一端を見るために2つ例題を見てみよう。

** $x$ 軸上の点電荷 [#ic5a4aaf]

$x$ 軸上に限り摩擦なく自由に動ける点電荷 $q$ (質量 $m$) が $t=0$ において $(-1,0)$ において静止している。$t>0$ においてこの点電荷が
$(0,1)$ に固定された点電荷 $-q$ からのクーロン力と、運動を $x$ 軸上に限るための拘束力のみを受けて運動するとして、点電荷の $x$ 座標の時間変化を表す運動方程式を求めよ。ただし系の誘電率を $\epsilon_0$ とする。

これまでと同様に普通にニュートン方程式を立てるなら、

電荷間の引力は $-\frac{1}{4\pi\epsilon_0}\frac{e^2}{x^2+1}$

その $x$ 軸に沿った成分は $-\frac{1}{4\pi\epsilon_0}\frac{e^2}{x^2+1}\frac{x}{\sqrt{x^2+1}}$

したがって、

$$
m\ddot x=-\frac{1}{4\pi\epsilon_0}\frac{e^2}{x^2+1}\frac{x}{\sqrt{x^2+1}}
$$

一方、ラグランジアンを使うと、

運動エネルギー $T=\frac{1}{2}m\dot x^2$

ポテンシャルエネルギー $U=\frac{1}{4\pi\epsilon_0}\frac{e^2}{\sqrt{x^2+1}}$

ラグランジアン $L=T-U$

$$
L=\frac{1}{2}m\dot x^2-\frac{1}{4\pi\epsilon_0}\frac{e^2}{\sqrt{x^2+1}}
$$

ラグランジュの運動方程式

$$
\frac{d}{dt}\left(\frac{\partial L}{\partial \dot x}\right)-\frac{\partial L}{\partial x}=0
$$

$$
m\ddot x=-\frac{e^2}{4\pi\epsilon_0}\frac{x}{(x^2+1)^{3/2}}
$$

として同じ式が得られる。

** 振り子の運動 [#sebbcf9b]

長さ $r$、重りの質量 $m$ の振り子について、その振れ角を $\theta$ として、$\theta$ に対する運動方程式を求めたい。

普通にやるなら重力加速度を $g$ として、

重力 $mg$

その円弧に沿った成分 $-mg\sin\theta$

円弧に沿った加速度 $r\ddot\theta$

より、運動方程式は

$$
mr\ddot\theta=-mg\sin\theta
$$

すなわち、

$$
\ddot\theta=-\frac gr\sin\theta
$$

一方、

重りの速度 $r\dot\theta$

高さは $r-r\cos\theta$ と表せるから、

運動エネルギー $T=\frac12 mr^2\dot\theta^2$

ポテンシャルエネルギー $U=mgr(1-\cos\theta)$

ラグランジアン $L=\frac12 mr^2\dot\theta^2-mgr(1-\cos\theta)$

$\theta$ に対する運動方程式は、

$$
\frac{d}{dt}\left\{\frac{\partial L}{\partial\dot\theta}\right\}=\frac{\partial L}{\partial\theta}
$$

$$
mr^2\ddot\theta=-mgr\sin\theta
$$

すなわち、

$$
\ddot\theta=-\frac{g}{r}\sin\theta
$$

を得る。

** 運動をラグランジアン形式で記述することのメリット [#z9cb4f3d]

ラグランジアンを用いた方法では力の働く方向などを考える必要がなく、
ラグランジアンさえ求めてしまえばそれを形式的に微分するのみで
運動方程式が得られる。

また、運動を記述するのが $x,y,z$ 座標であろうが、$r,\theta,\phi$ であろうが、
基本方程式が同じ形で記述できること(運動方程式が座標変換に対して共変である、という)
も非常に重要である。

* ラグランジュの運動方程式の共変性 [#kf999ae0]

ラグランジュの運動方程式の共変性は、その元となった最小作用の法則が座標の取り方に寄らない形式になっているため求めるまでもなく当然なりたつのであるが、ここでは練習も兼ねて明示的に示しておく。

座標系 $q_1,q_2,\dots,q_n$ から別の座標系 $Q_1,Q_2,\dots,Q_n$ への変数変換が、

$$
Q_i=Q_i(q_1,q_2,\dots,q_n)\ \ \ \ \ (i=1,2,\dots,n)
$$

で与えられるとする。またこの変換には逆変換が存在して、

$$
q_i=q_i(Q_1,Q_2,\dots,Q_n)\ \ \ \ \ (i=1,2,\dots,n)
$$

とする。

この座標変換は $x,y,z$ から $r,\theta,\phi$ のような幾何学的な座標変換であっても良いし、
2つの粒子の座標を重心座標と相対座標とに分けたり、あるいは
バネで連結された多数の質点の個々の位置座標から基準振動の振幅と位相へ変換するなど、どんな変換でも構わない。

座標 $q_1,q_2,\dots,q_n$ を決めれば座標 $Q_1,Q_2,\dots,Q_n$ を求められ、
逆に座標 $Q_1,Q_2,\dots,Q_n$ を決めれば座標 $q_1,q_2,\dots,q_n$ を求められる、
ということだけが座標変換の条件である。

このとき、すべての $i=1,2,\dots,n$ に対して、

$$
\frac{d}{dt}\left\{\frac{\partial L}{\partial \dot q_i}\right\}-\frac{\partial L}{\partial q_i}=0
$$

が成り立つことを仮定すると、すべての $i=1,2,\dots,n$ に対して、

$$
\frac{d}{dt}\left\{\frac{\partial L}{\partial \dot Q_i}\right\}-\frac{\partial L}{\partial Q_i}=0
$$

が成り立つことが以下のように導ける。(この話は $q_i$ と $Q_i$ に対して対称なので、当然逆も成り立つ)

まず、$L$ の $Q_i,\dot Q_i$ に対する偏微分を求めよう。

$$
\frac{\partial L}{\partial Q_i}
=\sum_{j=1}^n\frac{\partial q_j}{\partial Q_i}\frac{\partial L}{\partial q_j}+\sum_{j=1}^n\frac{\partial\dot q_j}{\partial Q_i}\frac{\partial L}{\partial\dot q_j}
$$

$$
\begin{aligned}
\frac{\partial L}{\partial\dot Q_i}
&=\sum_{j=1}^n\cancel\frac{\partial q_j}{\partial\dot Q_i}\frac{\partial L}{\partial q_j}+\sum_{j=1}^n\frac{\partial\dot q_j}{\partial\dot Q_i}\frac{\partial L}{\partial\dot q_j}\\
&=\sum_{j=1}^n\frac{\partial q_j}{\partial Q_i}\frac{\partial L}{\partial\dot q_j}\\
\end{aligned}
$$

第2式の変形は以下の理由で行える。

今、$\dot Q_j$ と $Q_j$ とは独立の変数と考えているため、$q_i$ は $Q_j$ の関数ではあっても $\dot Q_j$ の関数ではなく、$\frac{\partial q_j}{\partial\dot Q_i}=0$ である。また、


$$
\dot q_i=\sum_{j=1}^n \frac{\partial q_i}{\partial Q_j}\dot Q_j
$$

より、

$$
\frac{\partial\dot q_i}{\partial\dot Q_j}=\frac{\partial q_i}{\partial Q_j}
$$

であるのだ。

上式を代入すると、

$$
\begin{aligned}
\frac{d}{dt}\left\{\frac{\partial L}{\partial \dot Q_i}\right\}-\frac{\partial L}{\partial Q_i}
&=\sum_{j=1}^n\left[\frac{d}{dt}\left\{\frac{\partial q_j}{\partial Q_i}\frac{\partial L}{\partial \dot q_j}\right\}-\frac{\partial q_j}{\partial Q_i}\frac{\partial L}{\partial q_j}-\frac{\partial\dot q_j}{\partial Q_i}\frac{\partial L}{\partial\dot q_j}\right]\\
&=\sum_{j=1}^n\bigg[\frac{d}{dt}\left\{\frac{\partial q_j}{\partial Q_i}\right\}\frac{\partial L}{\partial \dot q_j}+\underbrace{\frac{\partial q_j}{\partial Q_i}\frac{d}{dt}\left\{\frac{\partial L}{\partial \dot q_j}\right\}-\frac{\partial q_j}{\partial Q_i}\frac{\partial L}{\partial q_j}}_{q_j,\dot q_j\text{に対する運動方程式よりゼロ}}-\frac{\partial\dot q_j}{\partial Q_i}\frac{\partial L}{\partial\dot q_j}\bigg]\\
&=\sum_{j=1}^n\bigg[\frac{\partial }{\partial Q_i}\frac{d}{dt}q_j-\frac{\partial\dot q_j}{\partial Q_i}\bigg]\frac{\partial L}{\partial \dot q_j}=0\\
\end{aligned}
$$

最下行冒頭の等号では $Q_i$ による偏微分と時間による全微分を入れ替えた。

* ラグランジアンの不定性 [#mcce419c]

ある系のラグランジアン $L$ に、粒子の位置と時間の関数 $F(q_1(t),q_2(t),\dots,q_n(t),t)$ の時間に対する全微分 $\frac{dF}{dt}$ を加えても運動方程式は変化しない。なぜなら、

$$
L'=L+\frac{dF}{dt}
$$

に対して、

$$
\begin{aligned}
S&=\int_{t_1}^{t_2}L'\ dt=\int_{t_1}^{t_2}\left(L+\frac{dF}{dt}\right)\ dt\\
&=\int_{t_1}^{t_2}L\ dt+\underbrace{F(q_1(t_2),q_2(t_2),\dots,q_n(t_2),t_2)-F(q_1(t_1),q_2(t_1),\dots,q_n(t_1),t_1)}_\text{定数}
\end{aligned}
$$

となって、$q_k(t_1),q_k(t_2)$ が固定されている限り $F(t_2)-F(t_1)$ は定数であり、
$S$ の $q_k$ による汎関数微分に影響を与えないためである。

したがって、ある力学系のラグランジアンを決定する際には上述の「運動量とポテンシャルによる定義」を離れ、 $F$ の分だけの不定性が残されていると考えてもよいことになる。
* 質問・コメント [#r1984971]

#article_kcaptcha

Counter: 44503 (from 2010/06/03), today: 10, yesterday: 0